Show that if $(A+2I)^2=0$, then $A+lambda I$ is invertible for $lambda ne 2$.
$begingroup$
Show that if $(A+2I)^2=0$, then $A+lambda I$ is invertible for $lambda ne 2$.
I tried to solve this by treating $(A+lambda I)v=0$ as linear equation system, and proving that $v$ must be $0$ (trivial solution) therefore $A+lambda I$ echelon form is I and it's invertible..
Would love to hear another solutions, and tips to my own proof:
$$A+lambda I=A+2I+(lambda-2) I.$$
$$(A+lambda I)v=0 Rightarrow (A+2I)v+(lambda-2)Iv=0.$$
$$(A+2I)(A+2I)v+(A+2I)(lambda-2)Iv=0 Rightarrow (lambda-2)(A+2I)Iv=0 Rightarrow (A+2I)Iv=0$$ (multiply by $A+2I$ and we know $lambda ne 2$).
$$(A+2I)v+(lambda-2)Iv=0 wedge (A+2I)Iv=0Rightarrow (λ−2)Iv=0 Rightarrow v=0$$
Therefore if v is solution for $(A+lambda I)v=0$ it must be 0.
(We have also shown that for $lambda = 2$, $v$ can be $(A+2I)$ which shows $A+2I$ isn't invertible)
BTW If A is such that $(A+2I)^2=0$, prove that $A+I$ is invertible. solves the basic case for $lambda = 1$
linear-algebra matrices eigenvalues-eigenvectors inverse matrix-equations
$endgroup$
|
show 1 more comment
$begingroup$
Show that if $(A+2I)^2=0$, then $A+lambda I$ is invertible for $lambda ne 2$.
I tried to solve this by treating $(A+lambda I)v=0$ as linear equation system, and proving that $v$ must be $0$ (trivial solution) therefore $A+lambda I$ echelon form is I and it's invertible..
Would love to hear another solutions, and tips to my own proof:
$$A+lambda I=A+2I+(lambda-2) I.$$
$$(A+lambda I)v=0 Rightarrow (A+2I)v+(lambda-2)Iv=0.$$
$$(A+2I)(A+2I)v+(A+2I)(lambda-2)Iv=0 Rightarrow (lambda-2)(A+2I)Iv=0 Rightarrow (A+2I)Iv=0$$ (multiply by $A+2I$ and we know $lambda ne 2$).
$$(A+2I)v+(lambda-2)Iv=0 wedge (A+2I)Iv=0Rightarrow (λ−2)Iv=0 Rightarrow v=0$$
Therefore if v is solution for $(A+lambda I)v=0$ it must be 0.
(We have also shown that for $lambda = 2$, $v$ can be $(A+2I)$ which shows $A+2I$ isn't invertible)
BTW If A is such that $(A+2I)^2=0$, prove that $A+I$ is invertible. solves the basic case for $lambda = 1$
linear-algebra matrices eigenvalues-eigenvectors inverse matrix-equations
$endgroup$
1
$begingroup$
I think you meant $lambda =1$ in your last sentence. Also what does "$v$ can be $A+2I$" mean ? I though $v$ was a vector ?
$endgroup$
– nicomezi
Dec 14 '18 at 12:14
1
$begingroup$
One can simplify a bit and consider $B = A+2I$. The problem then becomes "$B^2 = 0$ implies $B+lambda I$ invertible for $lambdaneq 0$".
$endgroup$
– Arthur
Dec 14 '18 at 12:14
$begingroup$
Hint: use @Arthur 's hint and use the Neumann series.
$endgroup$
– Stockfish
Dec 14 '18 at 12:16
$begingroup$
@Stockfish I get Arthur's hint but I am not sure I understood how to use Neumann series in this case
$endgroup$
– BBLN
Dec 14 '18 at 12:20
1
$begingroup$
Your proof is correct, I think that it's the simplest possible without using determinant.
$endgroup$
– Crostul
Dec 14 '18 at 12:53
|
show 1 more comment
$begingroup$
Show that if $(A+2I)^2=0$, then $A+lambda I$ is invertible for $lambda ne 2$.
I tried to solve this by treating $(A+lambda I)v=0$ as linear equation system, and proving that $v$ must be $0$ (trivial solution) therefore $A+lambda I$ echelon form is I and it's invertible..
Would love to hear another solutions, and tips to my own proof:
$$A+lambda I=A+2I+(lambda-2) I.$$
$$(A+lambda I)v=0 Rightarrow (A+2I)v+(lambda-2)Iv=0.$$
$$(A+2I)(A+2I)v+(A+2I)(lambda-2)Iv=0 Rightarrow (lambda-2)(A+2I)Iv=0 Rightarrow (A+2I)Iv=0$$ (multiply by $A+2I$ and we know $lambda ne 2$).
$$(A+2I)v+(lambda-2)Iv=0 wedge (A+2I)Iv=0Rightarrow (λ−2)Iv=0 Rightarrow v=0$$
Therefore if v is solution for $(A+lambda I)v=0$ it must be 0.
(We have also shown that for $lambda = 2$, $v$ can be $(A+2I)$ which shows $A+2I$ isn't invertible)
BTW If A is such that $(A+2I)^2=0$, prove that $A+I$ is invertible. solves the basic case for $lambda = 1$
linear-algebra matrices eigenvalues-eigenvectors inverse matrix-equations
$endgroup$
Show that if $(A+2I)^2=0$, then $A+lambda I$ is invertible for $lambda ne 2$.
I tried to solve this by treating $(A+lambda I)v=0$ as linear equation system, and proving that $v$ must be $0$ (trivial solution) therefore $A+lambda I$ echelon form is I and it's invertible..
Would love to hear another solutions, and tips to my own proof:
$$A+lambda I=A+2I+(lambda-2) I.$$
$$(A+lambda I)v=0 Rightarrow (A+2I)v+(lambda-2)Iv=0.$$
$$(A+2I)(A+2I)v+(A+2I)(lambda-2)Iv=0 Rightarrow (lambda-2)(A+2I)Iv=0 Rightarrow (A+2I)Iv=0$$ (multiply by $A+2I$ and we know $lambda ne 2$).
$$(A+2I)v+(lambda-2)Iv=0 wedge (A+2I)Iv=0Rightarrow (λ−2)Iv=0 Rightarrow v=0$$
Therefore if v is solution for $(A+lambda I)v=0$ it must be 0.
(We have also shown that for $lambda = 2$, $v$ can be $(A+2I)$ which shows $A+2I$ isn't invertible)
BTW If A is such that $(A+2I)^2=0$, prove that $A+I$ is invertible. solves the basic case for $lambda = 1$
linear-algebra matrices eigenvalues-eigenvectors inverse matrix-equations
linear-algebra matrices eigenvalues-eigenvectors inverse matrix-equations
edited Dec 14 '18 at 13:07
Batominovski
33k33293
33k33293
asked Dec 14 '18 at 12:09
BBLNBBLN
1214
1214
1
$begingroup$
I think you meant $lambda =1$ in your last sentence. Also what does "$v$ can be $A+2I$" mean ? I though $v$ was a vector ?
$endgroup$
– nicomezi
Dec 14 '18 at 12:14
1
$begingroup$
One can simplify a bit and consider $B = A+2I$. The problem then becomes "$B^2 = 0$ implies $B+lambda I$ invertible for $lambdaneq 0$".
$endgroup$
– Arthur
Dec 14 '18 at 12:14
$begingroup$
Hint: use @Arthur 's hint and use the Neumann series.
$endgroup$
– Stockfish
Dec 14 '18 at 12:16
$begingroup$
@Stockfish I get Arthur's hint but I am not sure I understood how to use Neumann series in this case
$endgroup$
– BBLN
Dec 14 '18 at 12:20
1
$begingroup$
Your proof is correct, I think that it's the simplest possible without using determinant.
$endgroup$
– Crostul
Dec 14 '18 at 12:53
|
show 1 more comment
1
$begingroup$
I think you meant $lambda =1$ in your last sentence. Also what does "$v$ can be $A+2I$" mean ? I though $v$ was a vector ?
$endgroup$
– nicomezi
Dec 14 '18 at 12:14
1
$begingroup$
One can simplify a bit and consider $B = A+2I$. The problem then becomes "$B^2 = 0$ implies $B+lambda I$ invertible for $lambdaneq 0$".
$endgroup$
– Arthur
Dec 14 '18 at 12:14
$begingroup$
Hint: use @Arthur 's hint and use the Neumann series.
$endgroup$
– Stockfish
Dec 14 '18 at 12:16
$begingroup$
@Stockfish I get Arthur's hint but I am not sure I understood how to use Neumann series in this case
$endgroup$
– BBLN
Dec 14 '18 at 12:20
1
$begingroup$
Your proof is correct, I think that it's the simplest possible without using determinant.
$endgroup$
– Crostul
Dec 14 '18 at 12:53
1
1
$begingroup$
I think you meant $lambda =1$ in your last sentence. Also what does "$v$ can be $A+2I$" mean ? I though $v$ was a vector ?
$endgroup$
– nicomezi
Dec 14 '18 at 12:14
$begingroup$
I think you meant $lambda =1$ in your last sentence. Also what does "$v$ can be $A+2I$" mean ? I though $v$ was a vector ?
$endgroup$
– nicomezi
Dec 14 '18 at 12:14
1
1
$begingroup$
One can simplify a bit and consider $B = A+2I$. The problem then becomes "$B^2 = 0$ implies $B+lambda I$ invertible for $lambdaneq 0$".
$endgroup$
– Arthur
Dec 14 '18 at 12:14
$begingroup$
One can simplify a bit and consider $B = A+2I$. The problem then becomes "$B^2 = 0$ implies $B+lambda I$ invertible for $lambdaneq 0$".
$endgroup$
– Arthur
Dec 14 '18 at 12:14
$begingroup$
Hint: use @Arthur 's hint and use the Neumann series.
$endgroup$
– Stockfish
Dec 14 '18 at 12:16
$begingroup$
Hint: use @Arthur 's hint and use the Neumann series.
$endgroup$
– Stockfish
Dec 14 '18 at 12:16
$begingroup$
@Stockfish I get Arthur's hint but I am not sure I understood how to use Neumann series in this case
$endgroup$
– BBLN
Dec 14 '18 at 12:20
$begingroup$
@Stockfish I get Arthur's hint but I am not sure I understood how to use Neumann series in this case
$endgroup$
– BBLN
Dec 14 '18 at 12:20
1
1
$begingroup$
Your proof is correct, I think that it's the simplest possible without using determinant.
$endgroup$
– Crostul
Dec 14 '18 at 12:53
$begingroup$
Your proof is correct, I think that it's the simplest possible without using determinant.
$endgroup$
– Crostul
Dec 14 '18 at 12:53
|
show 1 more comment
2 Answers
2
active
oldest
votes
$begingroup$
Below is a more general statement. I have provided three solutions to your question. The first two use the proposition, and the last one is as you requested.
Proposition. Let $n$ be positive integer and $A$ an $n$-by-$n$ matrix over a field $mathbb{K}$. Write $I$ for the $n$-by-$n$ identity matrix. For $muin K$, the matrix $A-mu, I$ is not invertible if and only if $mu$ is an eigenvalue of $A$ (or equivalently, $det(A-mu, I)=0$, or $ker(A-mu,I)neq {0}$).
Proof. For each $muin mathbb{K}$, let $V_muin mathbb{K}^n$ denote the kernel (i.e., the nullspace) of $A-mu,I$. If $A-mu,I$ is invertible, then for any $vin V_mu$, we have $(A-mu,I),v=0$ and so $$v=(A-mu,I)^{-1}(0)=0,,$$ implying that $V_mu={0}$. Conversely, if $V_mu={0}$, then $A-mu,I$ is an injective linear map from $mathbb{K}^n$ to itself. It is well known that any injective linear map on a finite-dimensional vector space is also surjective, whence bijective and so invertible. (This well known result is not true for infinite-dimensional vector spaces, by the way.) Therefore, $A-mu,I$ is invertible.
First Solution.
In your case, the only eigenvalue of $A$ is $-2$. Thus, $A-mu,I$ is invertible if and only if $muneq -2$, which is equivalent to saying that $A+lambda, I$ is invertible if and only if $lambdaneq 2$.
Second Solution.
We shall prove that $ker(A+lambda,I)={0}$ when $lambdaneq 2$. Suppose $vin ker(A+lambda,I)$. Then, $(A+lambda,I),v=0$, so $Av=-lambda,v$ and $A^2v=A(Av)=A(-lambda,v)=-lambda,(Av)=-lambda,(-lambda v)=lambda^2,v$. Since $(A+2,I)^2=0$, we also have $(A+2,I)^2,v=0$. Therefore, $$lambda^2,v-4,lambda,v+4,v=A^2v+4,Av,+4,v=0,.$$
Ergo, $(lambda-2)^2,v=0$. Since $lambdaneq 2$, $v=0$, which implies $ker(A+lambda ,I)={0}$.
Third Solution.
Since $lambda neq 2$, we have
$$frac{(x+2)^2-(x+4-lambda)(x+lambda)}{(lambda-2)^2}=1,,$$
where $x$ is a dummy variable. Therefore, $$frac{(A+2,I)^2-big(A+(4-lambda),Ibig),(A+lambda , I)}{(lambda-2)^2}=I,.$$
As $(A+2,I)^2=0$, we get
$$left(-frac{1}{(lambda-2)^2},big(A+(4-lambda),Ibig)right),(A+lambda,I)=I,.$$ Thence, $A+lambda,I$ is invertible and
$$(A+lambda,I)^{-1}=-frac{1}{(lambda-2)^2},big(A+(4-lambda),Ibig),.$$
$endgroup$
$begingroup$
I'm afraid I cant use eigenvalue or determinant. Basically we are only allowed to use "linear equation systems and trivial solution" or showing clearly a matrix B so that $(A+lambda I)B=I$
$endgroup$
– BBLN
Dec 14 '18 at 12:28
$begingroup$
@BBLN I wrote a proof which does not use determinant or eigenvalues.
$endgroup$
– Batominovski
Dec 14 '18 at 12:47
add a comment |
$begingroup$
Have you heard about eigenvalues? YOur equation for $A$ shows that $-2$ is the only eigenvalue of A(because any eigenvalue would satisfy $(x+2)^2=0$, the equation satisfied by $A$), so no other $lambda$ could be an eigenvalue for $A$, qed.
$endgroup$
add a comment |
Your Answer
StackExchange.ifUsing("editor", function () {
return StackExchange.using("mathjaxEditing", function () {
StackExchange.MarkdownEditor.creationCallbacks.add(function (editor, postfix) {
StackExchange.mathjaxEditing.prepareWmdForMathJax(editor, postfix, [["$", "$"], ["\\(","\\)"]]);
});
});
}, "mathjax-editing");
StackExchange.ready(function() {
var channelOptions = {
tags: "".split(" "),
id: "69"
};
initTagRenderer("".split(" "), "".split(" "), channelOptions);
StackExchange.using("externalEditor", function() {
// Have to fire editor after snippets, if snippets enabled
if (StackExchange.settings.snippets.snippetsEnabled) {
StackExchange.using("snippets", function() {
createEditor();
});
}
else {
createEditor();
}
});
function createEditor() {
StackExchange.prepareEditor({
heartbeatType: 'answer',
autoActivateHeartbeat: false,
convertImagesToLinks: true,
noModals: true,
showLowRepImageUploadWarning: true,
reputationToPostImages: 10,
bindNavPrevention: true,
postfix: "",
imageUploader: {
brandingHtml: "Powered by u003ca class="icon-imgur-white" href="https://imgur.com/"u003eu003c/au003e",
contentPolicyHtml: "User contributions licensed under u003ca href="https://creativecommons.org/licenses/by-sa/3.0/"u003ecc by-sa 3.0 with attribution requiredu003c/au003e u003ca href="https://stackoverflow.com/legal/content-policy"u003e(content policy)u003c/au003e",
allowUrls: true
},
noCode: true, onDemand: true,
discardSelector: ".discard-answer"
,immediatelyShowMarkdownHelp:true
});
}
});
Sign up or log in
StackExchange.ready(function () {
StackExchange.helpers.onClickDraftSave('#login-link');
});
Sign up using Google
Sign up using Facebook
Sign up using Email and Password
Post as a guest
Required, but never shown
StackExchange.ready(
function () {
StackExchange.openid.initPostLogin('.new-post-login', 'https%3a%2f%2fmath.stackexchange.com%2fquestions%2f3039282%2fshow-that-if-a2i2-0-then-a-lambda-i-is-invertible-for-lambda-ne-2%23new-answer', 'question_page');
}
);
Post as a guest
Required, but never shown
2 Answers
2
active
oldest
votes
2 Answers
2
active
oldest
votes
active
oldest
votes
active
oldest
votes
$begingroup$
Below is a more general statement. I have provided three solutions to your question. The first two use the proposition, and the last one is as you requested.
Proposition. Let $n$ be positive integer and $A$ an $n$-by-$n$ matrix over a field $mathbb{K}$. Write $I$ for the $n$-by-$n$ identity matrix. For $muin K$, the matrix $A-mu, I$ is not invertible if and only if $mu$ is an eigenvalue of $A$ (or equivalently, $det(A-mu, I)=0$, or $ker(A-mu,I)neq {0}$).
Proof. For each $muin mathbb{K}$, let $V_muin mathbb{K}^n$ denote the kernel (i.e., the nullspace) of $A-mu,I$. If $A-mu,I$ is invertible, then for any $vin V_mu$, we have $(A-mu,I),v=0$ and so $$v=(A-mu,I)^{-1}(0)=0,,$$ implying that $V_mu={0}$. Conversely, if $V_mu={0}$, then $A-mu,I$ is an injective linear map from $mathbb{K}^n$ to itself. It is well known that any injective linear map on a finite-dimensional vector space is also surjective, whence bijective and so invertible. (This well known result is not true for infinite-dimensional vector spaces, by the way.) Therefore, $A-mu,I$ is invertible.
First Solution.
In your case, the only eigenvalue of $A$ is $-2$. Thus, $A-mu,I$ is invertible if and only if $muneq -2$, which is equivalent to saying that $A+lambda, I$ is invertible if and only if $lambdaneq 2$.
Second Solution.
We shall prove that $ker(A+lambda,I)={0}$ when $lambdaneq 2$. Suppose $vin ker(A+lambda,I)$. Then, $(A+lambda,I),v=0$, so $Av=-lambda,v$ and $A^2v=A(Av)=A(-lambda,v)=-lambda,(Av)=-lambda,(-lambda v)=lambda^2,v$. Since $(A+2,I)^2=0$, we also have $(A+2,I)^2,v=0$. Therefore, $$lambda^2,v-4,lambda,v+4,v=A^2v+4,Av,+4,v=0,.$$
Ergo, $(lambda-2)^2,v=0$. Since $lambdaneq 2$, $v=0$, which implies $ker(A+lambda ,I)={0}$.
Third Solution.
Since $lambda neq 2$, we have
$$frac{(x+2)^2-(x+4-lambda)(x+lambda)}{(lambda-2)^2}=1,,$$
where $x$ is a dummy variable. Therefore, $$frac{(A+2,I)^2-big(A+(4-lambda),Ibig),(A+lambda , I)}{(lambda-2)^2}=I,.$$
As $(A+2,I)^2=0$, we get
$$left(-frac{1}{(lambda-2)^2},big(A+(4-lambda),Ibig)right),(A+lambda,I)=I,.$$ Thence, $A+lambda,I$ is invertible and
$$(A+lambda,I)^{-1}=-frac{1}{(lambda-2)^2},big(A+(4-lambda),Ibig),.$$
$endgroup$
$begingroup$
I'm afraid I cant use eigenvalue or determinant. Basically we are only allowed to use "linear equation systems and trivial solution" or showing clearly a matrix B so that $(A+lambda I)B=I$
$endgroup$
– BBLN
Dec 14 '18 at 12:28
$begingroup$
@BBLN I wrote a proof which does not use determinant or eigenvalues.
$endgroup$
– Batominovski
Dec 14 '18 at 12:47
add a comment |
$begingroup$
Below is a more general statement. I have provided three solutions to your question. The first two use the proposition, and the last one is as you requested.
Proposition. Let $n$ be positive integer and $A$ an $n$-by-$n$ matrix over a field $mathbb{K}$. Write $I$ for the $n$-by-$n$ identity matrix. For $muin K$, the matrix $A-mu, I$ is not invertible if and only if $mu$ is an eigenvalue of $A$ (or equivalently, $det(A-mu, I)=0$, or $ker(A-mu,I)neq {0}$).
Proof. For each $muin mathbb{K}$, let $V_muin mathbb{K}^n$ denote the kernel (i.e., the nullspace) of $A-mu,I$. If $A-mu,I$ is invertible, then for any $vin V_mu$, we have $(A-mu,I),v=0$ and so $$v=(A-mu,I)^{-1}(0)=0,,$$ implying that $V_mu={0}$. Conversely, if $V_mu={0}$, then $A-mu,I$ is an injective linear map from $mathbb{K}^n$ to itself. It is well known that any injective linear map on a finite-dimensional vector space is also surjective, whence bijective and so invertible. (This well known result is not true for infinite-dimensional vector spaces, by the way.) Therefore, $A-mu,I$ is invertible.
First Solution.
In your case, the only eigenvalue of $A$ is $-2$. Thus, $A-mu,I$ is invertible if and only if $muneq -2$, which is equivalent to saying that $A+lambda, I$ is invertible if and only if $lambdaneq 2$.
Second Solution.
We shall prove that $ker(A+lambda,I)={0}$ when $lambdaneq 2$. Suppose $vin ker(A+lambda,I)$. Then, $(A+lambda,I),v=0$, so $Av=-lambda,v$ and $A^2v=A(Av)=A(-lambda,v)=-lambda,(Av)=-lambda,(-lambda v)=lambda^2,v$. Since $(A+2,I)^2=0$, we also have $(A+2,I)^2,v=0$. Therefore, $$lambda^2,v-4,lambda,v+4,v=A^2v+4,Av,+4,v=0,.$$
Ergo, $(lambda-2)^2,v=0$. Since $lambdaneq 2$, $v=0$, which implies $ker(A+lambda ,I)={0}$.
Third Solution.
Since $lambda neq 2$, we have
$$frac{(x+2)^2-(x+4-lambda)(x+lambda)}{(lambda-2)^2}=1,,$$
where $x$ is a dummy variable. Therefore, $$frac{(A+2,I)^2-big(A+(4-lambda),Ibig),(A+lambda , I)}{(lambda-2)^2}=I,.$$
As $(A+2,I)^2=0$, we get
$$left(-frac{1}{(lambda-2)^2},big(A+(4-lambda),Ibig)right),(A+lambda,I)=I,.$$ Thence, $A+lambda,I$ is invertible and
$$(A+lambda,I)^{-1}=-frac{1}{(lambda-2)^2},big(A+(4-lambda),Ibig),.$$
$endgroup$
$begingroup$
I'm afraid I cant use eigenvalue or determinant. Basically we are only allowed to use "linear equation systems and trivial solution" or showing clearly a matrix B so that $(A+lambda I)B=I$
$endgroup$
– BBLN
Dec 14 '18 at 12:28
$begingroup$
@BBLN I wrote a proof which does not use determinant or eigenvalues.
$endgroup$
– Batominovski
Dec 14 '18 at 12:47
add a comment |
$begingroup$
Below is a more general statement. I have provided three solutions to your question. The first two use the proposition, and the last one is as you requested.
Proposition. Let $n$ be positive integer and $A$ an $n$-by-$n$ matrix over a field $mathbb{K}$. Write $I$ for the $n$-by-$n$ identity matrix. For $muin K$, the matrix $A-mu, I$ is not invertible if and only if $mu$ is an eigenvalue of $A$ (or equivalently, $det(A-mu, I)=0$, or $ker(A-mu,I)neq {0}$).
Proof. For each $muin mathbb{K}$, let $V_muin mathbb{K}^n$ denote the kernel (i.e., the nullspace) of $A-mu,I$. If $A-mu,I$ is invertible, then for any $vin V_mu$, we have $(A-mu,I),v=0$ and so $$v=(A-mu,I)^{-1}(0)=0,,$$ implying that $V_mu={0}$. Conversely, if $V_mu={0}$, then $A-mu,I$ is an injective linear map from $mathbb{K}^n$ to itself. It is well known that any injective linear map on a finite-dimensional vector space is also surjective, whence bijective and so invertible. (This well known result is not true for infinite-dimensional vector spaces, by the way.) Therefore, $A-mu,I$ is invertible.
First Solution.
In your case, the only eigenvalue of $A$ is $-2$. Thus, $A-mu,I$ is invertible if and only if $muneq -2$, which is equivalent to saying that $A+lambda, I$ is invertible if and only if $lambdaneq 2$.
Second Solution.
We shall prove that $ker(A+lambda,I)={0}$ when $lambdaneq 2$. Suppose $vin ker(A+lambda,I)$. Then, $(A+lambda,I),v=0$, so $Av=-lambda,v$ and $A^2v=A(Av)=A(-lambda,v)=-lambda,(Av)=-lambda,(-lambda v)=lambda^2,v$. Since $(A+2,I)^2=0$, we also have $(A+2,I)^2,v=0$. Therefore, $$lambda^2,v-4,lambda,v+4,v=A^2v+4,Av,+4,v=0,.$$
Ergo, $(lambda-2)^2,v=0$. Since $lambdaneq 2$, $v=0$, which implies $ker(A+lambda ,I)={0}$.
Third Solution.
Since $lambda neq 2$, we have
$$frac{(x+2)^2-(x+4-lambda)(x+lambda)}{(lambda-2)^2}=1,,$$
where $x$ is a dummy variable. Therefore, $$frac{(A+2,I)^2-big(A+(4-lambda),Ibig),(A+lambda , I)}{(lambda-2)^2}=I,.$$
As $(A+2,I)^2=0$, we get
$$left(-frac{1}{(lambda-2)^2},big(A+(4-lambda),Ibig)right),(A+lambda,I)=I,.$$ Thence, $A+lambda,I$ is invertible and
$$(A+lambda,I)^{-1}=-frac{1}{(lambda-2)^2},big(A+(4-lambda),Ibig),.$$
$endgroup$
Below is a more general statement. I have provided three solutions to your question. The first two use the proposition, and the last one is as you requested.
Proposition. Let $n$ be positive integer and $A$ an $n$-by-$n$ matrix over a field $mathbb{K}$. Write $I$ for the $n$-by-$n$ identity matrix. For $muin K$, the matrix $A-mu, I$ is not invertible if and only if $mu$ is an eigenvalue of $A$ (or equivalently, $det(A-mu, I)=0$, or $ker(A-mu,I)neq {0}$).
Proof. For each $muin mathbb{K}$, let $V_muin mathbb{K}^n$ denote the kernel (i.e., the nullspace) of $A-mu,I$. If $A-mu,I$ is invertible, then for any $vin V_mu$, we have $(A-mu,I),v=0$ and so $$v=(A-mu,I)^{-1}(0)=0,,$$ implying that $V_mu={0}$. Conversely, if $V_mu={0}$, then $A-mu,I$ is an injective linear map from $mathbb{K}^n$ to itself. It is well known that any injective linear map on a finite-dimensional vector space is also surjective, whence bijective and so invertible. (This well known result is not true for infinite-dimensional vector spaces, by the way.) Therefore, $A-mu,I$ is invertible.
First Solution.
In your case, the only eigenvalue of $A$ is $-2$. Thus, $A-mu,I$ is invertible if and only if $muneq -2$, which is equivalent to saying that $A+lambda, I$ is invertible if and only if $lambdaneq 2$.
Second Solution.
We shall prove that $ker(A+lambda,I)={0}$ when $lambdaneq 2$. Suppose $vin ker(A+lambda,I)$. Then, $(A+lambda,I),v=0$, so $Av=-lambda,v$ and $A^2v=A(Av)=A(-lambda,v)=-lambda,(Av)=-lambda,(-lambda v)=lambda^2,v$. Since $(A+2,I)^2=0$, we also have $(A+2,I)^2,v=0$. Therefore, $$lambda^2,v-4,lambda,v+4,v=A^2v+4,Av,+4,v=0,.$$
Ergo, $(lambda-2)^2,v=0$. Since $lambdaneq 2$, $v=0$, which implies $ker(A+lambda ,I)={0}$.
Third Solution.
Since $lambda neq 2$, we have
$$frac{(x+2)^2-(x+4-lambda)(x+lambda)}{(lambda-2)^2}=1,,$$
where $x$ is a dummy variable. Therefore, $$frac{(A+2,I)^2-big(A+(4-lambda),Ibig),(A+lambda , I)}{(lambda-2)^2}=I,.$$
As $(A+2,I)^2=0$, we get
$$left(-frac{1}{(lambda-2)^2},big(A+(4-lambda),Ibig)right),(A+lambda,I)=I,.$$ Thence, $A+lambda,I$ is invertible and
$$(A+lambda,I)^{-1}=-frac{1}{(lambda-2)^2},big(A+(4-lambda),Ibig),.$$
edited Dec 14 '18 at 13:02
answered Dec 14 '18 at 12:25
BatominovskiBatominovski
33k33293
33k33293
$begingroup$
I'm afraid I cant use eigenvalue or determinant. Basically we are only allowed to use "linear equation systems and trivial solution" or showing clearly a matrix B so that $(A+lambda I)B=I$
$endgroup$
– BBLN
Dec 14 '18 at 12:28
$begingroup$
@BBLN I wrote a proof which does not use determinant or eigenvalues.
$endgroup$
– Batominovski
Dec 14 '18 at 12:47
add a comment |
$begingroup$
I'm afraid I cant use eigenvalue or determinant. Basically we are only allowed to use "linear equation systems and trivial solution" or showing clearly a matrix B so that $(A+lambda I)B=I$
$endgroup$
– BBLN
Dec 14 '18 at 12:28
$begingroup$
@BBLN I wrote a proof which does not use determinant or eigenvalues.
$endgroup$
– Batominovski
Dec 14 '18 at 12:47
$begingroup$
I'm afraid I cant use eigenvalue or determinant. Basically we are only allowed to use "linear equation systems and trivial solution" or showing clearly a matrix B so that $(A+lambda I)B=I$
$endgroup$
– BBLN
Dec 14 '18 at 12:28
$begingroup$
I'm afraid I cant use eigenvalue or determinant. Basically we are only allowed to use "linear equation systems and trivial solution" or showing clearly a matrix B so that $(A+lambda I)B=I$
$endgroup$
– BBLN
Dec 14 '18 at 12:28
$begingroup$
@BBLN I wrote a proof which does not use determinant or eigenvalues.
$endgroup$
– Batominovski
Dec 14 '18 at 12:47
$begingroup$
@BBLN I wrote a proof which does not use determinant or eigenvalues.
$endgroup$
– Batominovski
Dec 14 '18 at 12:47
add a comment |
$begingroup$
Have you heard about eigenvalues? YOur equation for $A$ shows that $-2$ is the only eigenvalue of A(because any eigenvalue would satisfy $(x+2)^2=0$, the equation satisfied by $A$), so no other $lambda$ could be an eigenvalue for $A$, qed.
$endgroup$
add a comment |
$begingroup$
Have you heard about eigenvalues? YOur equation for $A$ shows that $-2$ is the only eigenvalue of A(because any eigenvalue would satisfy $(x+2)^2=0$, the equation satisfied by $A$), so no other $lambda$ could be an eigenvalue for $A$, qed.
$endgroup$
add a comment |
$begingroup$
Have you heard about eigenvalues? YOur equation for $A$ shows that $-2$ is the only eigenvalue of A(because any eigenvalue would satisfy $(x+2)^2=0$, the equation satisfied by $A$), so no other $lambda$ could be an eigenvalue for $A$, qed.
$endgroup$
Have you heard about eigenvalues? YOur equation for $A$ shows that $-2$ is the only eigenvalue of A(because any eigenvalue would satisfy $(x+2)^2=0$, the equation satisfied by $A$), so no other $lambda$ could be an eigenvalue for $A$, qed.
answered Dec 14 '18 at 12:25
Sorin TircSorin Tirc
1,755213
1,755213
add a comment |
add a comment |
Thanks for contributing an answer to Mathematics Stack Exchange!
- Please be sure to answer the question. Provide details and share your research!
But avoid …
- Asking for help, clarification, or responding to other answers.
- Making statements based on opinion; back them up with references or personal experience.
Use MathJax to format equations. MathJax reference.
To learn more, see our tips on writing great answers.
Sign up or log in
StackExchange.ready(function () {
StackExchange.helpers.onClickDraftSave('#login-link');
});
Sign up using Google
Sign up using Facebook
Sign up using Email and Password
Post as a guest
Required, but never shown
StackExchange.ready(
function () {
StackExchange.openid.initPostLogin('.new-post-login', 'https%3a%2f%2fmath.stackexchange.com%2fquestions%2f3039282%2fshow-that-if-a2i2-0-then-a-lambda-i-is-invertible-for-lambda-ne-2%23new-answer', 'question_page');
}
);
Post as a guest
Required, but never shown
Sign up or log in
StackExchange.ready(function () {
StackExchange.helpers.onClickDraftSave('#login-link');
});
Sign up using Google
Sign up using Facebook
Sign up using Email and Password
Post as a guest
Required, but never shown
Sign up or log in
StackExchange.ready(function () {
StackExchange.helpers.onClickDraftSave('#login-link');
});
Sign up using Google
Sign up using Facebook
Sign up using Email and Password
Post as a guest
Required, but never shown
Sign up or log in
StackExchange.ready(function () {
StackExchange.helpers.onClickDraftSave('#login-link');
});
Sign up using Google
Sign up using Facebook
Sign up using Email and Password
Sign up using Google
Sign up using Facebook
Sign up using Email and Password
Post as a guest
Required, but never shown
Required, but never shown
Required, but never shown
Required, but never shown
Required, but never shown
Required, but never shown
Required, but never shown
Required, but never shown
Required, but never shown
1
$begingroup$
I think you meant $lambda =1$ in your last sentence. Also what does "$v$ can be $A+2I$" mean ? I though $v$ was a vector ?
$endgroup$
– nicomezi
Dec 14 '18 at 12:14
1
$begingroup$
One can simplify a bit and consider $B = A+2I$. The problem then becomes "$B^2 = 0$ implies $B+lambda I$ invertible for $lambdaneq 0$".
$endgroup$
– Arthur
Dec 14 '18 at 12:14
$begingroup$
Hint: use @Arthur 's hint and use the Neumann series.
$endgroup$
– Stockfish
Dec 14 '18 at 12:16
$begingroup$
@Stockfish I get Arthur's hint but I am not sure I understood how to use Neumann series in this case
$endgroup$
– BBLN
Dec 14 '18 at 12:20
1
$begingroup$
Your proof is correct, I think that it's the simplest possible without using determinant.
$endgroup$
– Crostul
Dec 14 '18 at 12:53